15

多重ゼータ値とその類似 Part4

1000
0
$$\newcommand{BA}[0]{\begin{align*}} \newcommand{bl}[0]{\boldsymbol} \newcommand{D}[0]{\displaystyle} \newcommand{Df}[0]{{\rm\mathbf{Definition.}}} \newcommand{EA}[0]{\end{align*}} \newcommand{k}[0]{\boldsymbol{k}} \newcommand{L}[0]{\left} \newcommand{l}[0]{\boldsymbol{l}} \newcommand{LA}[0]{\langle} \newcommand{m}[0]{\boldsymbol{m}} \newcommand{n}[0]{\boldsymbol{n}} \newcommand{Pr}[0]{{\rm\mathbf{Proof.}}} \newcommand{qed}[0]{\hspace{450pt}\Box} \newcommand{R}[0]{\right} \newcommand{RA}[0]{\rangle} \newcommand{Rem}[0]{{\rm\mathbf{Remarks.}}} $$

 
${\rm\bf{Lemma~4.1.}}$  $0< x\le 1$に対して

$ \D[4.1]   {\rm Li}_{{\{1\}}_{a-1},2}(1-x)=\zeta(a+1)-\sum_{j=0}^{a}\frac{1}{j!}\sum_{0< n}\frac{x^n}{n^{a-j+1}}\ln^{j}\frac{1}{x} $

 
$\Pr$

$\BA \D   {\rm Li}_{{\{1\}}_{a-1},2}(1-x) &=\int_0^{1-x} \frac{1}{1-t}\L(\circ\frac{1}{1-t}\R)^{a-1}\circ\frac{1}{t}\\ &=\int_x^1 \frac{1}{1-t}\L(\circ\frac{1}{t}\R)^a\\ &=\sum_{0< n}\int_x^1 t^{n-1}\L(\circ\frac{1}{t}\R)^a\\ &=\sum_{0< n}\int_x^1 \frac{1}{t}\frac{t^n-x^n}{n}\L(\circ\frac{1}{t}\R)^{a-1}\\ &=\sum_{0< n}\int_x^1 \frac{1}{t}\left(\frac{t^n-x^n}{n^2}-\frac{x^n}{n}\ln\frac{t}{x}\right)\L(\circ\frac{1}{t}\R)^{a-2}\\ &=\cdots\\ &=\sum_{0< n}\L(\frac{1}{n^{a+1}}-\sum_{j=0}^{a}\frac{1}{j!}\frac{x^n}{n^{a-j+1}}\ln^{j}\frac{1}{x}\R)\\ &=\zeta(a+1)-\sum_{j=0}^{a}\frac{1}{j!}\sum_{0< n}\frac{x^n}{n^{a-j+1}}\ln^{j}\frac{1}{x} \EA$
$\qed$

 
${\rm\bf{Conjecture~4.1.}}$  $a\in\mathbb{Z}_{>0},~b\in\mathbb{Z}_{\ge 0} $に対して

$ \D[4.2]   {\rm Li}_{{\{1\}}_{a-1},b+2}(1-x) =\sum_{0< n}\left(\frac{1}{n^{a+1}}-\sum_{j=0}^a\frac{x^n}{n^{a-j+1}} {\rm Li}_{{\{1\}}_{j}}(1-x) \right) \sum_{k=0}^{b}(-1)^k\zeta_{n-1}(\{1\}_{b-k}){\rm Li}_{{\{1\}}_k}(x) $

著者は$b=0,1,2$での証明に成功したが,一般の$b$での厳密な証明は未達成。計算方法は確立しているが。

 
${\rm\bf{Corollary~4.1.}}$  $x<1$に対して

$ \D[4.3]   \sum_{0< n}\frac{\zeta_{n-1}(\{1\}_{a-1})}{n^3}\frac{n!}{(1-x)_{n}} =\sum_{0< m\le n}\frac{1}{(m-x)n^{a+1}}-\sum_{j=1}^a \sum_{0< m< n}\frac{1}{m^{a-j+1}(n-x)^{j+1}} $

 
$\Pr$

$\BA \D   \sum_{0< n}\frac{\zeta_{n-1}(\{1\}_{a-1})}{n^3}\frac{n!}{(1-x)_{n}} &=\int_0^1 \frac{t^{-x}}{1-t}\,{\rm Li}_{{\{1\}}_{a-1},2}(1-t)\,dt\\ &=\int_0^1\frac{t^{-x}}{1-t}\sum_{0< n}\L(\frac{1-t^n}{n^{a+1}}-\sum_{j=1}^{a}\frac{1}{j!}\frac{t^n}{n^{a-j+1}}\ln^{j}\frac{1}{t} \R)dt\\ &=\sum_{0< n}\L(\frac{1}{n^{a+1}}\int_0^1 \frac{t^{-x}(1-t^n)}{1-t}\,dt-\sum_{j=1}^{a}\frac{1}{j!n^{a-j+1}}\int_0^1 \frac{t^{n-x}}{1-t}\ln^{j}\frac{1}{t} \R)\\ &=\sum_{0< n}\L(\frac{1}{n^{a+1}}\sum_{m=1}^{n}\int_0^1t^{m-1+x}dt-\sum_{j=1}^{a}\frac{1}{j!n^{a-j+1}}\sum_{n< m}\int_0^1 t^{m-1-x}\ln^j\frac{1}{t}\,dt \R)\\ &=\sum_{0< n}\L(\frac{1}{n^{a+1}}\sum_{m=1}^{n}\frac{1}{m-x}-\sum_{j=1}^{a}\frac{1}{j!n^{a-j+1}}\sum_{n< m}\frac{j!}{(m-x)^{j+1}} \R)\\ &=\sum_{0< m\le n}\frac{1}{(m-x)n^{a+1}}-\sum_{j=1}^a \sum_{0< m< n}\frac{1}{m^{a-j+1}(n-x)^{j+1}} \EA$
$\qed$

 
${\rm\bf{Corollary~4.2.}}$

$ \D[4.4]   \sum_{0< n}\frac{\zeta_{n-1}(\{1\}_{a-1}){\zeta}^{\star}_n(\{1\}_{b})}{n^3} =\zeta^{\star}(b+1,a+1)-\sum_{j=1}^a\binom{j+b-1}{b}\zeta(a-j+1,j+b+1) $

 
$\Pr$
 $[4.3]$において,$x$について冪級数展開して係数を比較する。
$\qed$

 
${\rm\bf{Theorem~4.1.}}$

$ \D[4.5]   {\rm Li}_{\{1\}_{a-1},b+1}(1-x) =\sum_{j=0}^{b-1}(-1)^j\zeta(\{1\}_{a-1},b-j+1){\rm Li}_{\{1\}_{j}}(x) +(-1)^b\sum_{0\le n_1\le \cdots\le n_b\le a}{\rm Li}_{\{1\}_{n_1}}(1-x)\,{\rm Li}_{a-n_b+1,n_b-n_{b-1}+1,\cdots,n_2-n_1+1}(x) $

 
$\Pr$

$ \D   \int_0^x \frac{1}{1-t}\,{\rm Li}_{{\{1\}}_{a-1},2}(1-t)\,dt=\zeta(\{1\}_{a-1},3)-{\rm Li}_{{\{1\}}_{a-1},3}(1-x) $

$\BA \D   \int_0^x \frac{1}{1-t}\,{\rm Li}_{{\{1\}}_{a-1},2}(1-t)\,dt &=\int_0^x \frac{1}{1-t}\L(\zeta(a+1)-\sum_{j=0}^{a}\frac{1}{j!}\sum_{0< n}\frac{t^n}{n^{a-j+1}}\ln^{j}\frac{1}{t}\R)dt\\ &=\zeta(a+1)\ln\frac{1}{1-x}-\sum_{j=0}^a\frac{1}{j!}\sum_{0< n}\frac{1}{n^{a-j+1}}\int_0^x \frac{t^n}{1-t}\ln^{j}\frac{1}{t}\,dt\\ &=\zeta(\{1\}_{a-1},2)\ln\frac{1}{1-x}-\sum_{j=0}^a\frac{1}{j!}\sum_{0< n}\frac{1}{n^{a-j+1}}\sum_{k=0}^j\frac{j!}{k!}\ln^k\frac{1}{x}\sum_{n< m}\frac{x^m}{m^{j-k+1}}\\ &=\zeta(\{1\}_{a-1},2)\ln\frac{1}{1-x}-\sum_{0\le k\le j\le a}\frac{1}{k!}\ln^k\frac{1}{x}\sum_{0< n< m}\frac{x^m}{n^{a-j+1}m^{j-k+1}} \EA$

$\D\int_0^x\frac{1}{1-t}\,dt$を繰り返せばよい。
$\qed$

投稿日:2021725

この本を高評価した人

高評価したユーザはいません

この本に送られたバッジ

バッジはありません。

投稿者

コメント

他の人のコメント

コメントはありません。
読み込み中...
読み込み中
前ページへ
多重ゼータ値とその類似の結果・導出法まとめの表紙
次ページへ